K
Khách

Hãy nhập câu hỏi của bạn vào đây, nếu là tài khoản VIP, bạn sẽ được ưu tiên trả lời.

17 tháng 11 2017

làm xong rồi thì please_sign

áp dụng bđt huyền thoại \(\frac{1}{a^2}+\frac{1}{b^2}+\frac{1}{c^2}\ge\frac{1}{ab}+\frac{1}{bc}+\frac{1}{ac}\) =\(\frac{a+b+c}{abc}=\frac{\left(a+b+c\right)^2}{abc\left(a+b+c\right)}\) 

mà \(\left(ab+bc+ac\right)^2\ge3abc\left(a+b+c\right)\) (tụ cm nhé )

\(\Rightarrow\ge\frac{\left(a+b+c^2\right)}{\frac{\left(ab+bc+ac\right)^2}{3}}=\frac{3\left(a+b+c\right)^2\left(a^2+b^2+c^2\right)}{\left(ab+bc+ac\right)^2\left(a^2+b^2+c^2\right)}\)

m,à \(\left(ab+bc+ac\right)^2\left(a^2+b^2+c^2\right)\le\frac{\left(a^2+b^2+c^2+ab+bc+ac+ab+bc+ac\right)^3}{3^3}\)

   =\(\frac{\left(\left(a+b+c\right)^2\right)^3}{27}=27\)

\(\Rightarrow vt\ge\frac{27\left(a^2+b^2+c^2\right)}{27}=a^2+b^2+c^2\)

dau = khi a=b=c=1

17 tháng 11 2017

hay quá bạn ơi

Câu 1: Cho \(a,b,c0\)và \(a+b+c=3\). Chứng minh rằng:\(\frac{a}{1+b^2}+\frac{b}{1+c^2}+\frac{c}{1+a^2}\ge\frac{3}{2}\).Câu 2: Cho \(a,b,c,d0\)và \(a+b+c+d=4\). Chứng minh rằng:\(\frac{a}{1+b^2}+\frac{b}{1+c^2}+\frac{c}{1+d^2}+\frac{d}{1+a^2}\ge2\).Câu 3: Cho \(a,b,c,d0\). Chứng minh rằng:\(\frac{a^3}{a^2+b^2}+\frac{b^3}{b^2+c^2}+\frac{c^3}{c^2+d^2}+\frac{d^3}{d^2+a^2}\ge\frac{a+b+c+d}{2}\).Câu 4: Cho \(a,b,c,d0\). Chứng minh...
Đọc tiếp

Câu 1Cho \(a,b,c>0\)và \(a+b+c=3\). Chứng minh rằng:

\(\frac{a}{1+b^2}+\frac{b}{1+c^2}+\frac{c}{1+a^2}\ge\frac{3}{2}\).

Câu 2: Cho \(a,b,c,d>0\)và \(a+b+c+d=4\). Chứng minh rằng:

\(\frac{a}{1+b^2}+\frac{b}{1+c^2}+\frac{c}{1+d^2}+\frac{d}{1+a^2}\ge2\).

Câu 3: Cho \(a,b,c,d>0\). Chứng minh rằng:

\(\frac{a^3}{a^2+b^2}+\frac{b^3}{b^2+c^2}+\frac{c^3}{c^2+d^2}+\frac{d^3}{d^2+a^2}\ge\frac{a+b+c+d}{2}\).

Câu 4: Cho \(a,b,c,d>0\). Chứng minh rằng:

\(\frac{a^4}{a^3+2b^3}+\frac{b^4}{b^3+2c^3}+\frac{c^4}{c^3+2d^3}+\frac{d^4}{d^3+2a^3}\ge\frac{a+b+c+d}{3}\).

Câu 5: Cho \(a,b,c>0\)và \(a+b+c=3\). Chứng minh rằng:

\(\frac{a^2}{a+2b^2}+\frac{b^2}{b+2c^2}+\frac{c^2}{c+2a^2}\ge1\).

Câu 6: Cho \(a,b,c>0\)và \(a+b+c=3\). Chứng minh rằng: 

\(\frac{a^2}{a+2b^3}+\frac{b^2}{b+2c^3}+\frac{c^2}{c+2a^3}\ge1\).

Câu 7: Cho \(a,b,c>0\)và \(a+b+c=3\). Chứng minh rằng:

\(\frac{a+1}{b^2+1}+\frac{b+1}{c^2+1}+\frac{c+1}{a^2+1}\ge3\).

Câu 8: Cho \(a_1,a_2,...,a_{n-1},a_n>0\)và \(a_1+a_2+...+a_{n-1}+a_n=n\)với \(n\)nguyên dương. Chứng minh:

\(\frac{1}{a_1+1}+\frac{1}{a_2+1}+...+\frac{1}{a_{n-1}+1}+\frac{1}{a_n+1}\ge\frac{n}{2}\).

 

 

0
15 tháng 11 2017

tịt ??????????????????????????????????????????????????______________________?????????????????????????????????????????????

15 tháng 11 2017

ai trả lời bài này hộ tui cái !!!

20 tháng 7 2016

Ta có : 

\(\sqrt{\frac{1}{a^2}+\frac{1}{b^2}+\frac{1}{c^2}}=\sqrt{\left(\frac{1}{a}+\frac{1}{b}+\frac{1}{c}\right)^2-\frac{2}{ab}-\frac{2}{bc}-\frac{2}{ca}}\)

                                   \(=\sqrt{\left(\frac{1}{a}+\frac{1}{b}+\frac{1}{c}\right)^2-2\left(\frac{a+b+c}{abc}\right)}\)

                                   \(=\sqrt{\left(\frac{1}{a}+\frac{1}{b}+\frac{1}{c}\right)^2}=\left[\frac{1}{a}+\frac{1}{b}+\frac{1}{c}\right]\left(ĐPCM\right)\)

[ ] là giá trị tuyệt đối đấy.

ủng hộ nhé bạn!

18 tháng 12 2019

\(VT=\frac{\left(\sqrt[3]{abc}\right)^2}{2abc}+\Sigma\frac{a^2}{a^2\left(b+c\right)}\ge\frac{\left(a+b+c+\sqrt[3]{abc}\right)^2}{\Sigma a^2\left(b+c\right)+2abc}=\frac{\left(a+b+c+\sqrt[3]{abc}\right)^2}{\left(a+b\right)\left(b+c\right)\left(c+a\right)}\)

9 tháng 8 2019

2) Theo nguyên lí Dirichlet, trong ba số \(a^2-1;b^2-1;c^2-1\) có ít nhất hai số nằm cùng phía với 1.

Giả sử đó là a2 - 1 và b2 - 1. Khi đó \(\left(a^2-1\right)\left(b^2-1\right)\ge0\Leftrightarrow a^2b^2-a^2-b^2+1\ge0\)

\(\Rightarrow a^2b^2+3a^2+3b^2+9\ge4a^2+4b^2+8\)

\(\Rightarrow\left(a^2+3\right)\left(b^2+3\right)\ge4\left(a^2+b^2+2\right)\)

\(\Rightarrow\left(a^2+3\right)\left(b^2+3\right)\left(c^2+3\right)\ge4\left(a^2+b^2+1+1\right)\left(1+1+c^2+1\right)\) (2)

Mà \(4\left[\left(a^2+b^2+1+1\right)\left(1+1+c^2+1\right)\right]\ge4\left(a+b+c+1\right)^2\) (3)(Áp dụng Bunhicopxki và cái ngoặc vuông)

Từ (2) và (3) ta có đpcm.

Sai thì chịu

9 tháng 8 2019

Xí quên bài 2 b:v

b) Không mất tính tổng quát, giả sử \(\left(a^2-\frac{1}{4}\right)\left(b^2-\frac{1}{4}\right)\ge0\)

Suy ra \(a^2b^2-\frac{1}{4}a^2-\frac{1}{4}b^2+\frac{1}{16}\ge0\)

\(\Rightarrow a^2b^2+a^2+b^2+1\ge\frac{5}{4}a^2+\frac{5}{4}b^2+\frac{15}{16}\)

Hay \(\left(a^2+1\right)\left(b^2+1\right)\ge\frac{5}{4}\left(a^2+b^2+\frac{3}{4}\right)\)

Suy ra \(\left(a^2+1\right)\left(b^2+1\right)\left(c^2+1\right)\ge\frac{5}{4}\left(a^2+b^2+\frac{1}{4}+\frac{1}{2}\right)\left(\frac{1}{4}+\frac{1}{4}+c^2+\frac{1}{2}\right)\)

\(\ge\frac{5}{4}\left(\frac{1}{2}a+\frac{1}{2}b+\frac{1}{2}c+\frac{1}{2}\right)^2=\frac{5}{16}\left(a+b+c+1\right)^2\) (Bunhiacopxki) (đpcm)

Đẳng thức xảy ra khi \(a=b=c=\frac{1}{2}\)

22 tháng 3 2019

Áp dụng bất đẳng thức Cô-si ta có:

\(\dfrac{a^2}{b^3}+\dfrac{1}{a}+\dfrac{1}{a}\ge\sqrt[3]{\dfrac{a^2}{b^3}.\dfrac{1}{a}.\dfrac{1}{a}}=\dfrac{3}{b}\)

\(\dfrac{c^2}{a^3}+\dfrac{1}{c}+\dfrac{1}{c}\ge\sqrt[3]{\dfrac{c^2}{a^3}.\dfrac{1}{c}.\dfrac{1}{c}}=\dfrac{3}{a}\)

\(\dfrac{c^2}{a^3}+\dfrac{1}{c}+\dfrac{1}{c}\ge\sqrt[3]{\dfrac{c^2}{a^3}.\dfrac{1}{c}.\dfrac{1}{c}}=\dfrac{3}{a}\)

Cộng theo vế ta được:

\(\dfrac{a^2}{b^3}+\dfrac{b^2}{c^3}+\dfrac{a^2}{a^3}+\dfrac{2}{a}+\dfrac{2}{b}+\dfrac{2}{c}\ge3\left(\dfrac{1}{a}+\dfrac{1}{b}+\dfrac{1}{c}\right)\)

\(\Leftrightarrow\dfrac{a^2}{b^3}+\dfrac{b^2}{c^3}+\dfrac{c^2}{a^3}\ge\dfrac{1}{a}+\dfrac{1}{b}+\dfrac{1}{c}\)

8 tháng 5 2018

\(\frac{a^2}{b^3}+\frac{b^2}{c^3}+\frac{c^2}{a^3}=\frac{1}{b}+\frac{1}{c}+\frac{1}{a}\)

=> \(\frac{a^2}{b^3}+\frac{b^2}{c^3}+\frac{c^2}{a^3}=\frac{1}{a}+\frac{1}{b}+\frac{1}{c}\)

20 tháng 5 2018

\(\frac{a^2}{b^3}+\frac{1}{a}+\frac{1}{a}\ge3\cdot\frac{1}{b}\)

28 tháng 11 2019

Bài này đăng nhiều trên OLM rồi, lời giải vắn tắt:

\(VT=\Sigma_{cyc}\frac{a}{1+b^2}=\Sigma_{cyc}\left(a-\frac{ab^2}{1+b^2}\right)=3-\Sigma_{cyc}\frac{ab^2}{1+b^2}\)

\(\ge3-\Sigma_{cyc}\frac{ab}{2}\ge3-\frac{\frac{\left(a+b+c\right)^2}{3}}{2}=\frac{3}{2}\)

Đẳng thức xảy ra khi a = b = c = 1

28 tháng 11 2019

Ta có: \(\frac{a}{1+b^2}=a-\frac{ab^2}{1+b^2}\ge a-\frac{ab^2}{2b}=a-\frac{ab}{2}\)(bđt cô - si)

Tương tự ta có: \(\frac{b}{1+c^2}\ge b-\frac{bc}{2}\);\(\frac{c}{1+a^2}\ge c-\frac{ca}{2}\)

Cộng từng vế của các bđt trên:

\(\frac{a}{1+b^2}\)\(+\frac{b}{1+c^2}\)\(+\frac{c}{1+a^2}\)\(\ge a+b+c-\frac{ab+bc+ca}{2}\)

Dễ c/m:  \(\left(a+b+c\right)^2\ge3\left(ab+bc+ca\right)\)

\(\Rightarrow3^2\ge3\left(ab+bc+ca\right)\)

\(\Rightarrow ab+bc+ca\le3\)

\(BĐT\ge3-\frac{3}{2}=\frac{3}{2}\)

hay \(\frac{a}{1+b^2}\)\(+\frac{b}{1+c^2}\)\(+\frac{c}{1+a^2}\)\(\ge\frac{3}{2}\)

(Dấu "="\(\Leftrightarrow a=b=1\))

4 tháng 9 2016

Bạn có thể tham khảo cách này

Đặt \(\hept{\begin{cases}\frac{1}{a}=x\\\frac{2}{b}=y\\\frac{3}{c}=z\end{cases}}\Rightarrow x+y+z=3\)

BĐT thành \(\frac{x^3}{x^2+y^2}+\frac{y^3}{y^2+z^2}+\frac{z^3}{z^2+x^2}\ge\frac{3}{2}\left(1\right)\)

ta sẽ dùng Bđt Cói \(\frac{x^3}{x^2+y^2}=x-\frac{xy^2}{x^2+y^2}\ge x-\frac{xy^2}{2xy}=x-\frac{y}{2}\)

Tương tự rồi cộng lại

\(\left(1\right)\ge x+y+z-\frac{x+y+z}{2}=3-\frac{3}{2}=\frac{3}{2}\)

Dấu = khi \(x=y=z=1\Rightarrow\hept{\begin{cases}a=1\\b=2\\c=3\end{cases}}\)

4 tháng 9 2016

Đặt \(\hept{\begin{cases}x=\frac{1}{a}\\y=\frac{2}{b}\\z=\frac{3}{c}\end{cases}\Rightarrow}\hept{\begin{cases}x,y,z>0\\x+y+z=3\end{cases}}\)

Khi đó ta có BĐT cần chứng minh tương đương với:

\(P=\frac{x^3}{x^2+y^2}+\frac{y^3}{y^2+z^2}+\frac{z^3}{z^2+x^2}\ge\frac{3}{2}\)

Ta có: \(P\ge\frac{\left(x^2+y^2+z^2\right)^2}{x^2y+y^2z+z^2x+xy^2+yz^2+zx^2}\)

Ta cũng có: \(3\left(x^2+y^2+z^2\right)=\left(x+y+z\right)\left(x^2+y^2+z^2\right)\)

\(=x^3+y^3+z^3+xy^2+yz^2+zx^2+x^2y+y^2z+z^2x\)

\(\ge3\left(x^2y+y^2z+z^2x\right)\)

\(\Rightarrow x^2y+y^2z+z^2x\le x^2+y^2+z^2\)

Chứng minh tương tự ta có: \(xy^2+yz^2+zx^2\le x^2+y^2+z^2\)

\(\Rightarrow P\ge\frac{x^2+y^2+z^2}{2}\ge\frac{\left(x+y+z\right)^2}{3}=\frac{3}{2}\)

Dấu = khi \(x=y=z\)hay\(\hept{\begin{cases}a=1\\b=2\\b=3\end{cases}}\)